Question

In: Economics

In the short run a firm has noticed that the price of their most important fixed...

In the short run a firm has noticed that the price of their most important fixed input has risen. What impact will this have on profit? Why? What if this increase in price had occurred in the long run? Why is this different than in the short run?

Solutions

Expert Solution

The increase in fixed cost like lease payment, insurance, salaries and interest on borrowed fund etc increases the average total cost of the firms in shortrun. The increase in total cost will reduce the profit of the firm. If the price is equal to average total cost, the firm start to incur loss. If the market price is above the ATC in shortrun the increase in fixed cost will lowers the profit of the firm.

In longrun the price is equal to ATC. Thus the firms are in break even or earning zero economic profit. No price other than a price equal to ATC exists in longrun due to the free entry and exit of the firms. In shortrun if the fixed cost increase, the firms will quit the industry in longrun and the price will increase due to the fall in supply out of the exit of the firm. So the loss in shortrun due to increased fixed cost will be fully covered by the increase in price to the level of ATC.


Related Solutions

A perfectly competitive firm has the following fixed and variable costs in the short run. The market price for the firm’s product is $150.
A perfectly competitive firm has the following fixed and variable costs in the short run. The market price for the firm’s product is $150.Complete the table.OutputFCVCTCMCTRMRProfit/Loss0$100$01100100210018031003004100440   5100600   6100780   At what output rate does the firm maximize profit or minimize loss?What is the firm’s marginal revenue at each positive level of output? Its average revenue?What can you say about the relationship between marginal revenue and marginal cost for output rates below the profit –maximizing (or loss minimizing) rate? For output rates above the profit...
Is ? or ? fixed in the short-run?
Is ? or ? fixed in the short-run?
Question 4: A firm has the following short-run production functions. The total fixed cost of the...
Question 4: A firm has the following short-run production functions. The total fixed cost of the firm is AED 1,800. The firm pays a wage rate of AED 300 per day for each worker. The firm’s only variable cost is the wages of workers. Calculate the following:            A. Calculate the Average Product of Labor and the Marginal Product of Labor by filling the empty cells in the following table                                                             (0.5x10=5 Marks)                                                                                                                                                                                                                                          Number of Workers Total Output (Product)       (per day) Average...
At a price of $10, and assuming the price doesn't increase in the future, should the firm continue to produce in the short-run or shut down in the short-run?
Quantity Total Revenue Marginal Revenue Total Cost Marginal Cost Fixed Costs ATC Average Fixed Costs Average Variable Costs 0 0 - 10 - 10 - - - 1 8 24 14 24 2 16 34 10 17 3 24 42 8 14 4 32 49 7 12.25 5 40 57 8 11.4 6 48 67 10 11.17 7 56 81 14 11.57 8 64 99 18 12.38 9 72 123 24 13.67 At a price of $10, and assuming the...
how a fixed factor's price effect the variable factor and the profit in short run?
how a fixed factor's price effect the variable factor and the profit in short run?
The supply curve of a price-taker firm in the short run is the: Group of answer...
The supply curve of a price-taker firm in the short run is the: Group of answer choices firm's average variable cost curve. portion of the firm's average total cost curve that lies above average variable cost curve. portion of the firm's marginal cost curve that lies above average variable cost curve. firm's marginal revenue curve.
(1) Will the firm produce in the short-run at the price? Of $41? Explain why?
Quantity, QAFCAVCATCMCPAR=PTR=P*QMR=P056016045.00105.0045.005656565623042.5072.5040.0056561125632040.0060.0035.0056561685641537.5052.5030.0056562245651237.0049.0035.0056562805661037.5047.5040.0056563365678.5738.5747.1445.0056563925687.540.6348.1355.0056564485696.6743.3350.0065.0056565045610646.5052.5075.00565656056(b). Now assume that product price falls to $41.(1) Will the firm produce in the short-run at the price? Of $41? Explain why?(2) If yes, how many units? Is this output profit-maximizing or loss minimizing? What would be profit/loss? (show all your calculations). Show the firm equilibrium on a separate graph using average cost curves.(c). Assume price drops further to $32.(1) Will this firm produce or not? Explain if not, why not? Determine profit/losses and show your answer graphically using average/marginal...
A stock trader at a brokerage firm has noticed that price changes in the shares of...
A stock trader at a brokerage firm has noticed that price changes in the shares of Summer House Bakery each trade are dependent upon the previous trade's price change. Her observations can be summarized by the following transition matrix. (We are assuming +1/8, 0, -1/8 are the only possible price changes for this stock). ​ Next Price Change Most Recent Price Change +1/8 0 -1/8 +1/8 .7 .2 .1 0 .3 .4 .3 -1/8 .2 .1 .7 ​A) Provide a...
A stock trader at a brokerage firm has noticed that price changes in the shares of...
A stock trader at a brokerage firm has noticed that price changes in the shares of Summer House Bakery each trade are dependent upon the previous trade's price change. Her observations can be summarized by the following transition matrix. (We are assuming +1/8, 0, -1/8 are the only possible price changes for this stock). ​ Next Price Change Most Recent Price Change +1/8 0 -1/8 +1/8 .7 .2 .1 0 .3 .4 .3 -1/8 .2 .1 .7 ​A) Provide a...
A profit-maximizing firm in a perfectly competitive market operates in the short run with total fixed...
A profit-maximizing firm in a perfectly competitive market operates in the short run with total fixed costs of D $2,250 and total variable costs (TVC) as is below. The firm can only produce integer amounts of output (Q): Q (Output) TVC (Total Variable Cost 0 0.00 1 2,500 2 4,000 3 5,000 4 6,200 5 7,600 6 9,360 7 11,500 8 13,860 9 16,450 10 19,200 11 22,310 3a. How much output should the firm produce if it can sell...
ADVERTISEMENT
ADVERTISEMENT
ADVERTISEMENT